It’s easy but...

Algebra Level 3

Given that f ( x + y ) = f ( x ) + f ( y ) + 2 x y f(x+y)=f(x)+f(y)+2xy , if f ( 10 ) = 135 f(10)=135 , find f ( 1 2 ) f\left(\frac{1}{2}\right) .


The answer is 2.

This section requires Javascript.
You are seeing this because something didn't load right. We suggest you, (a) try refreshing the page, (b) enabling javascript if it is disabled on your browser and, finally, (c) loading the non-javascript version of this page . We're sorry about the hassle.

2 solutions

Brian Moehring
Sep 25, 2018

Note that g ( x ) : = f ( x ) x 2 g ( x + y ) = g ( x ) + g ( y ) [ 1 ] g ( x ) = C x for some C g(x) := f(x) - x^2 \implies g(x+y) = g(x) + g(y) \iff\!\!\!^{\color{#3D99F6}[1]}\,\,\, g(x) = C\cdot x \quad \text{ for some } C so that f ( x ) = x 2 + C x f(x) = x^2 + C\cdot x for some C . C.

Now set x = 10 x=10 : 135 = f ( 10 ) = 1 0 2 + 10 C C = 3.5 135 = f(10) = 10^2 + 10C \implies C = 3.5 so that f ( 1 2 ) = ( 1 2 ) 2 + 3.5 ( 1 2 ) = 2 f\left(\tfrac{1}{2}\right) = \left(\tfrac{1}{2}\right)^2 + 3.5\left(\tfrac{1}{2}\right) = \boxed{2}


Remark on [ 1 ] ^{\color{#3D99F6}[1]} : This step (specifically the "only if" direction) actually assumes g g (and therefore f f ) is continuous at zero. This isn't necessarily the case, but if it bothers you, simply replace f f with f Q f|_\mathbb{Q} by restricting its domain to the rational numbers.

Tom Engelsman
Sep 26, 2018

Let's try a calculus approach to this functional equation. Integrating it with respect to x x and y y respectively yields:

f ( x + y ) = f ( x ) + 2 y f'(x+y) = f'(x) + 2y AND f ( x + y ) = f ( y ) + 2 x f'(x+y) = f'(y) + 2x

which if we equate these two above equations, we obtain:

f ( x ) + 2 y = f ( y ) + 2 x f ( x ) = 2 x + [ f ( y ) 2 y ] = 2 x + A f'(x) + 2y = f'(y) + 2x \Rightarrow f'(x) = 2x + [f'(y) - 2y] = 2x + A (for A R ) . A \in \mathbb{R}). (i).

Integrating (i) produces:

f ( x ) = x 2 + A x + B f(x) = x^2 + Ax + B (ii).

Substituting (ii) back into the original functional equation now yields:

( x + y ) 2 + A ( x + y ) + B = ( x 2 + A x + B ) + ( y 2 + A y + B ) + 2 x y x 2 + 2 x y + y 2 + A ( x + y ) + B = x 2 + 2 x y + y 2 + A ( x + y ) + 2 B (x+y)^2 + A(x+y) + B = (x^2 + Ax + B) + (y^2 + Ay + B) + 2xy \Rightarrow x^2 + 2xy + y^2 + A(x+y) + B = x^2 + 2xy + y^2 + A(x+y) + 2B (iii)

Upon observation, we must have B = 0 B = 0 in order for (iii) to hold for all x , y R . x,y \in \mathbb{R}. Using the provided initial condition f ( 10 ) = 135 f(10) = 135 , we now solve for A A :

135 = 1 0 2 + 10 A A = 3.5 135 = 10^2 + 10A \Rightarrow A = 3.5

which leaves us with f ( x ) = x 2 + 3.5 x f(x) = x^2 + 3.5x , and f ( 1 2 ) = ( 1 2 ) 2 + 3.5 ( 1 2 ) = 2 . f(\frac{1}{2}) = (\frac{1}{2})^2 + 3.5(\frac{1}{2}) = \boxed{2}.

0 pending reports

×

Problem Loading...

Note Loading...

Set Loading...